K
Khách

Hãy nhập câu hỏi của bạn vào đây, nếu là tài khoản VIP, bạn sẽ được ưu tiên trả lời.

6 tháng 12 2019

\(\frac{a^3}{\left(b+1\right)\left(c+1\right)}+\frac{b+1}{8}+\frac{c+1}{8}\ge\frac{3}{4}a\)\(\Leftrightarrow\)\(\frac{a^3}{\left(b+1\right)\left(c+1\right)}\ge\frac{3}{4}a-\frac{1}{8}b-\frac{1}{8}-\frac{1}{4}\)

\(\Sigma\frac{a^3}{\left(b+1\right)\left(c+1\right)}\ge\frac{1}{2}\left(a+b+c\right)-\frac{3}{4}\ge\frac{3}{2}-\frac{3}{4}=\frac{3}{4}\) :) 

23 tháng 11 2020

1)

Ta có: \(M=\Sigma_{cyc}\frac{\sqrt{3}\left(a+b+4c\right)}{\sqrt{3\left(a+b\right)\left(a+b+4c\right)}}\ge\Sigma_{cyc}\frac{\sqrt{3}\left(a+b+4c\right)}{\frac{3\left(a+b\right)+\left(a+b+4c\right)}{2}}=\Sigma_{cyc}\frac{\sqrt{3}\left(a+b+4c\right)}{2\left(a+b+c\right)}=3\sqrt{3}\)

Dấu "=" xảy ra khi a=b=c

24 tháng 11 2020

2)

\(\Sigma_{cyc}\sqrt[3]{\left(\frac{2a}{ab+1}\right)^2}=\Sigma_{cyc}\frac{2a}{\sqrt[3]{2a\left(ab+1\right)^2}}\ge\Sigma_{cyc}\frac{2a}{\frac{2a+\left(ab+1\right)+\left(ab+1\right)}{3}}=3\Sigma_{cyc}\frac{a}{ab+a+1}\)

Ta có bổ đề: \(\frac{a}{ab+a+1}+\frac{b}{bc+b+1}+\frac{c}{ca+c+1}=1\left(abc=1\right)\)

\(\Rightarrow\Sigma_{cyc}\sqrt[3]{\left(\frac{2a}{ab+1}\right)^2}\ge3\)

8 tháng 7 2016

bài 2 thì bạn áp dụng bdt cô si với lựa chọn điểm rơi  hoặc bdt holder  ( nó giống kiểu bunhia ngược ) . bai 1 thi ap dung cai nay \(\frac{1}{x}+\frac{1}{y}>=\frac{1}{x+y}\)  câu 1 khó hơn nhưng bạn biết lựa chọn điểm rơi với áp dụng bdt phụ kia là ok .

9 tháng 7 2016

Bài 1:Đặt VT=A

Dùng BĐT \(\left(x+y+z\right)\left(\frac{1}{x}+\frac{1}{y}+\frac{1}{z}\right)\ge9\Rightarrow\frac{1}{x+y+z}\le\frac{1}{9}\left(\frac{1}{x}+\frac{1}{y}+\frac{1}{z}\right)x,y,z>0\)

Áp dụng vào bài toán trên với x=a+c;y=b+a;z=2b ta có:

\(\frac{ab}{a+3b+2c}=\frac{ab}{\left(a+c\right)+\left(b+c\right)+2b}\le\frac{ab}{9}\left(\frac{1}{a+c}+\frac{1}{b+c}+\frac{1}{2b}\right)\)

Tương tự với 2 cái còn lại

\(A\le\frac{1}{9}\left(\frac{bc+ac}{a+b}+\frac{bc+ab}{a+c}+\frac{ab+ac}{b+c}\right)+\frac{1}{18}\left(a+b+c\right)\)

\(\Rightarrow A\le\frac{1}{9}\left(a+b+c\right)+\frac{1}{18}\left(a+b+c\right)=\frac{a+b+c}{6}\)

Đẳng thức xảy ra khi a=b=c 

Bài 2:

Biến đổi BPT \(4\left(\frac{a^3}{\left(1+b\right)\left(1+c\right)}+\frac{b^3}{\left(1+c\right)\left(1+a\right)}+\frac{c^3}{\left(1+a\right)\left(1+b\right)}\right)\ge3\)

\(\Rightarrow\frac{a^3}{\left(1+b\right)\left(1+c\right)}+\frac{b^3}{\left(1+c\right)\left(1+a\right)}+\frac{c^3}{\left(1+a\right)\left(1+b\right)}\ge\frac{3}{4}\)

Dự đoán điểm rơi xảy ra khi a=b=c=1

\(\frac{a^3}{\left(1+b\right)\left(1+c\right)}+\frac{1+b}{8}+\frac{1+c}{8}\ge\frac{3a}{4}\)

Tương tự suy ra

\(VT\ge\frac{2\left(a+b+c\right)-3}{4}\ge\frac{2\cdot3\sqrt{abc}-3}{4}=\frac{3}{4}\)

23 tháng 8 2017

Bài này làm hoài :v

Áp dụng BĐT Cauchy-Schwarz và BĐT AM-GM ta có:

\(VT=\frac{b^2c^2}{ab+ac}+\frac{a^2c^2}{ab+bc}+\frac{a^2b^2}{ac+bc}\)

\(\ge\frac{\left(ab+bc+ca\right)^2}{2\left(ab+bc+ca\right)}=\frac{ab+bc+ca}{2}\)

\(\ge\frac{3\sqrt[3]{\left(abc\right)^2}}{2}=\frac{3}{2}=VP\)

Khi a=b=c=1

2 tháng 9 2020

Đặt \(\left\{a;b;c\right\}\rightarrow\left\{\frac{1}{x};\frac{1}{y};\frac{1}{z}\right\}\)Khi đó : \(\frac{1}{x}.\frac{1}{y}.\frac{1}{z}=\frac{1}{x.y.z}=a.b.c=1< =>x.y.z=1\)

\(BĐT< =>\frac{1}{\left(\frac{1}{x}\right)^3\left(\frac{1}{y}+\frac{1}{z}\right)}+\frac{1}{\left(\frac{1}{y}\right)^3\left(\frac{1}{y}+\frac{1}{x}\right)}+\frac{1}{\left(\frac{1}{z}\right)^3\left(\frac{1}{x}+\frac{1}{y}\right)}\ge\frac{3}{2}\)

\(< =>\frac{x^3yz}{y+z}+\frac{y^3xz}{z+x}+\frac{z^3xy}{x+y}\ge\frac{3}{2}\)\(< =>\frac{x^2}{y+z}+\frac{y^2}{x+z}+\frac{z^2}{x+y}\ge\frac{3}{2}\)(*)

Ta chỉ cần chỉ ra bất đẳng thức (*) đúng thì bài toán được giải quyết , thật vậy :

Theo bất đẳng thức Bunhiacopxki dạng phân thức :

\(\frac{x^2}{y+z}+\frac{y^2}{z+x}+\frac{z^2}{x+y}\ge\frac{\left(x+y+z\right)^2}{y+z+z+x+x+y}=\frac{\left(x+y+z\right)^2}{2\left(x+y+z\right)}=\frac{x+y+z}{2}\) (**)

Sử dụng bất đẳng thức AM-GM ta có : 

\(x+y+z\ge3\sqrt[3]{xyz}=3\sqrt[3]{1}=3\)Tương đương \(\frac{x+y+z}{2}\ge\frac{3}{2}\)(***)

Từ (**) và (***) ta được \(\frac{x^2}{z+y}+\frac{y^2}{x+z}+\frac{z^2}{x+y}\ge\frac{x+y+z}{2}\ge\frac{3}{2}\)

Suy ra bất đẳng thức (*) đúng . Nên ta có điều phải chứng minh !

Dấu "=" xảy ra khi và chỉ khi \(x=y=z=1< =>a=b=c=1\)

15 tháng 11 2017

ta có: \(\frac{a}{\left(a+1\right)\left(b+1\right)}+\frac{b}{\left(b+1\right)\left(c+1\right)}+\frac{c}{\left(c+1\right)\left(a+1\right)}.\)

\(\ge3\sqrt[3]{\frac{a.b.c}{\left(a+1\right)^2.\left(b+1\right)^2.\left(c+1\right)^2}}=\frac{3}{\sqrt[3]{\left(a+1\right)^2.\left(b+1\right)^2.\left(c+1\right)^2}}\)    (vì abc=1)     (*)

Mặt khác: \(\left(a+1\right)^2.\left(b+1\right)^2.\left(c+1\right)^2\ge64abc=64=4^3\)   (vì abc=1)

=> \(\sqrt[3]{\left(a+1\right)^2.\left(b+1\right)^2.\left(c+1\right)^2}\ge4\)   (**)

Từ (*), (**)=> đpcm

12 tháng 2 2020

Bạn dưới kia làm ngược dấu thì phải,mà bài này hình như là mũ 3

\(\frac{a^3}{\left(a+1\right)\left(b+1\right)}+\frac{a+1}{8}+\frac{b+1}{8}\ge3\sqrt[3]{\frac{a^3\left(a+1\right)\left(b+1\right)}{64\left(a+1\right)\left(b+1\right)}}=\frac{3a}{4}\)

Tương tự rồi cộng lại:

\(RHS+\frac{2\left(a+b+c\right)+6}{8}\ge\frac{3\left(a+b+c\right)}{4}\)

\(\Leftrightarrow RHS\ge\frac{3}{4}\) tại a=b=c=1

10 tháng 2 2018

Từ giả thiết của bài toán, ta biến đổi như sau:

\(a^2+b^2+c^2+\left(a+b+c\right)^2\le4\)

\(\Leftrightarrow a^2+b^2+c^2+ab+ac+bc\le2\)
Bất đẳng thức cần chứng minh tương đương với

\(A=\frac{ab+1}{\left(a+b\right)^2}+\frac{bc+1}{\left(b+c\right)^2}+\frac{ac+1}{\left(a+c\right)^2}\ge3\)

\(\Leftrightarrow\frac{2ab+2}{\left(a+b\right)^2}+\frac{2bc+2}{\left(b+c\right)^2}+\frac{2ac+2}{\left(a+c\right)^2}\ge6\)
Áp dụng giả thiết ta được

\(\frac{2ab+2}{\left(a+b\right)^2}+\frac{2ab+2}{\left(b+c\right)^2}+\frac{2ac+2}{\left(a+c\right)^2}\ge\text{∑}\frac{2ab+a^2+b^2+c^2+ab+bc+ac}{\left(a+b\right)^2}\)

\(=1+\frac{\left(c+a\right)\left(c+b\right)}{\left(a+b\right)^2}+1+\frac{\left(b+a\right)\left(c+b\right)}{\left(a+c^2\right)}+1+\frac{\left(c+a\right)\left(a+b\right)}{\left(c+b\right)^2}\)

\(=3+\frac{\left(c+a\right)\left(c+b\right)}{\left(a+b\right)^2}+\frac{\left(b+a\right)\left(c+b\right)}{\left(a+c\right)^2}+\frac{\left(c+a\right)\left(a+b\right)}{\left(c+b\right)^2}\ge\)

\(3+\sqrt[3]{\frac{\left(c+a\right)\left(c+b\right)\left(b+a\right)\left(c+b\right)\left(c+a\right)\left(a+b\right)}{\left[\left(a+b\right)\left(b+c\right)\left(c+a\right)\right]^2}}=3+3=6\)



Vậy bài toán đã được chứng minh. Đẳng thức xảy ra khi và chỉ khi a=b=c=13√.■

28 tháng 8 2020

Áp dụng giả thiết và một đánh giá quen thuộc, ta được: \(16\left(a+b+c\right)\ge\frac{1}{a}+\frac{1}{b}+\frac{1}{c}=\frac{ab+bc+ca}{abc}=\frac{\left(ab+bc+ca\right)^2}{abc\left(ab+bc+ca\right)}\ge\frac{3\left(a+b+c\right)}{ab+bc+ca}\)hay \(\frac{1}{6\left(ab+bc+ca\right)}\le\frac{8}{9}\)

Đến đây, ta cần chứng minh \(\frac{1}{\left(a+b+\sqrt{2\left(a+c\right)}\right)^3}+\frac{1}{\left(b+c+\sqrt{2\left(b+a\right)}\right)^3}+\frac{1}{\left(c+a+\sqrt{2\left(c+b\right)}\right)^3}\le\frac{1}{6\left(ab+bc+ca\right)}\)

 Áp dụng bất đẳng thức Cauchy cho ba số dương ta có \(a+b+\sqrt{2\left(a+c\right)}=a+b+\sqrt{\frac{a+c}{2}}+\sqrt{\frac{a+c}{2}}\ge3\sqrt[3]{\frac{\left(a+b\right)\left(a+c\right)}{2}}\)hay \(\left(a+b+\sqrt{2\left(a+c\right)}\right)^3\ge\frac{27\left(a+b\right)\left(a+c\right)}{2}\Leftrightarrow\frac{1}{\left(a+b+2\sqrt{a+c}\right)^3}\le\frac{2}{27\left(a+b\right)\left(a+c\right)}\)

Hoàn toàn tương tự ta có \(\frac{1}{\left(b+c+2\sqrt{b+a}\right)^3}\le\frac{2}{27\left(b+c\right)\left(b+a\right)}\)\(\frac{1}{\left(c+a+2\sqrt{c+b}\right)^3}\le\frac{2}{27\left(c+a\right)\left(c+b\right)}\)

Cộng theo vế các bất đẳng thức trên ta được \(\frac{1}{\left(a+b+\sqrt{2\left(a+c\right)}\right)^3}+\frac{1}{\left(b+c+\sqrt{2\left(b+a\right)}\right)^3}+\frac{1}{\left(c+a+\sqrt{2\left(c+b\right)}\right)^3}\le\frac{4\left(a+b+c\right)}{27\left(a+b\right)\left(b+c\right)\left(c+a\right)}\)Phép chứng minh sẽ hoàn tất nếu ta chỉ ra được \(\frac{4\left(a+b+c\right)}{27\left(a+b\right)\left(b+c\right)\left(c+a\right)}\le\frac{1}{6\left(ab+bc+ca\right)}\)\(\Leftrightarrow\left(a+b\right)\left(b+c\right)\left(c+a\right)\ge\frac{8}{9}\left(ab+bc+ca\right)\left(a+b+c\right)\)

Đây là một đánh giá đúng, thật vậy: đặt a + b + c = p; ab + bc + ca = q; abc = r thì bất đẳng thức trên trở thành \(pq-r\ge\frac{8}{9}pq\Leftrightarrow\frac{1}{9}pq\ge r\)*đúng vì \(a+b+c\ge3\sqrt[3]{abc}\)\(ab+bc+ca\ge3\sqrt[3]{\left(abc\right)^2}\))

Vậy bất đẳng thức được chứng minh

Đẳng thức xảy ra khi \(a=b=c=\frac{1}{4}\)

6 tháng 1 2018

sửa giả thiết là \(\left(ab\right)^3+\left(bc\right)^3+\left(ca\right)^3=3\left(abc\right)^2\)

Và Áp dụng BĐT cô-si, ta có \(\left(ab\right)^3+\left(bc\right)^3+\left(ca\right)^3\ge3\left(abc\right)^2\)

dấu = xảy ra <=>a=b=c>0

Thay vào thì \(\left(1+\frac{a}{b}\right)\left(1+\frac{b}{c}\right)\left(1+\frac{c}{a}\right)=8\) (ĐPCM)

^_^